Electromagnetic Field Lagrangian - Field Equations

In summary: Correct. So you get \frac{\partial L}{\partial \left(\partial_{\beta}A_{\alpha}\right)} = \frac{1}{8\pi}\left[\delta^{\beta}_{\mu}\delta^{\alpha}_{\nu} \partial_{\mu}A_{\nu} - \delta^{\beta}_{\nu}\delta^{\alpha}_{\mu} \partial_{\mu}A_{\nu}\right] Now in the second term in the bracket ##\beta## and ##\alpha## are summed over so that the term becomes ##\frac{\partial\left(\partial^{\beta}A^{\alpha}\right)}{\partial \left
  • #1
ProfDawgstein
80
1
I was working on an exercise in Ohanian's book.

[Appendix A3, page 484, Exercise 5]

Show that ##\mathcal{L}_{em}## leads to the field equation ##\partial_\mu \partial^\mu A^\nu - \partial^\nu \partial_\mu A^\mu = 4\pi j^\nu## with ##[\partial_\nu] j^\nu = 0##.

I guess he means charge conservation, but wrote ##j^\nu = 0##.

The Lagrangian was given by ##\mathcal{L}_{em} = -\frac{1}{16\pi} \left( A_{\mu ,\nu} - A_{\nu ,\mu} \right) \left( A^{\mu ,\nu} - A^{\nu ,\mu} \right)##.

Which should be the same as ##\mathcal{L}_{em} = -\frac{1}{16\pi} F_{\mu\nu} F^{\mu\nu}##.

The Euler-Lagrange Equation is ##\frac{\partial}{\partial x^\mu} \frac{\partial \mathcal{L}}{\partial A^{\nu}_{\ ,\mu}} - \frac{\partial \mathcal{L}}{\partial A^\nu} = 0##.

##\frac{\partial \mathcal{L}}{\partial A^\nu} = 0## because there are no ##A^\nu## terms.

Now, because there are different components (covariant, contravariant), I insert some ##\eta_{\mu\nu}## to make them all look the same.
-> making the derivatives easier later

This leads to ##\mathcal{L} = -\frac{1}{16\pi} (\eta_{\mu\alpha} A^{\alpha}_{\ ,\nu} - \eta_{\alpha\nu} A^{\alpha}_{\ ,\mu}) (\eta^{\beta\nu} A^{\mu}_{\ ,\beta} - \eta^{\beta\mu} A^{\nu}_{\ ,\beta})##.

Now just multiply...

Then I get this

##\mathcal{L} = -\frac{1}{16\pi} \eta_{\mu\alpha} \eta^{\beta\nu} A^{\alpha}_{\ ,\nu} A^{\mu}_{\ ,\beta} + ... + ... - ...##

Now I am supposed to do

##\frac{\partial \mathcal{L}}{\partial A^{\nu}_{\ ,\mu}}##

which means

##-\frac{1}{16\pi} \eta_{\mu\alpha} \eta^{\beta\nu} \frac{\partial}{\partial A^{\nu}_{\ ,\mu}} (A^{\alpha}_{\ ,\nu} A^{\mu}_{\ ,\beta})##

now product rule

...

then I get terms like this

##\frac{\partial A^{\alpha}_{\ ,\nu}}{\partial A^{\nu}_{\ ,\mu}}##

My problem:

1) what about the indices?
-do I need to rename all of ##{\partial A^{\nu}_{\ ,\mu}}## or just the ##,\mu##?

2) what do I get out of that?
-my thoughts were something like ##\delta^a_b##

What I did was

##\frac{\partial A^{\alpha}_{\ ,\nu}}{\partial A^{\nu}_{\ ,\mu}} = \delta^\mu_\nu##

-ignoring the top (contravariant) indices
-just using the ##,\mu## and ##,\nu##

Thank you in advance.

I would be grateful if somebody could show the result of
##-\frac{1}{16\pi} \eta_{\mu\alpha} \eta^{\beta\nu} \frac{\partial}{\partial A^{\nu}_{\ ,\mu}} (A^{\alpha}_{\ ,\nu} A^{\mu}_{\ ,\beta})##
 
Physics news on Phys.org
  • #2
Perhaps a simpler example will get you up to speed. Consider the Klein-Gordon Lagrangian density in flat space-time ##\mathcal{L} = -\frac{1}{2}(\eta^{ab}\partial_{a}\varphi \partial_{b}\varphi + m^2 \varphi^2)##.

We have ##\frac{\partial
\mathcal{L}}{\partial \varphi} = -m^{2}\varphi## and ##\frac{\partial\mathcal{L}}{\partial(\partial_{a} \varphi)} = -\frac{1}{2}\{\eta^{bc}\partial_{b}\varphi \frac{\partial (\partial_{c}\varphi)}{\partial (\partial_{a}\varphi)} + \eta^{bc}\partial_{c}\varphi \frac{\partial (\partial_{b}\varphi)}{\partial (\partial_{a}\varphi)}\}\\ = -\frac{1}{2}\{\eta^{bc}\partial_{b}\varphi\delta^{a}_{c} + \eta^{bc}\partial_{c}\varphi \delta^{a}_{b}\}\\ = -\partial^{a}\varphi##
where ##\frac{\partial (\partial_{a}\varphi)}{\partial (\partial_{b}\varphi)} = \delta^{b}_{a}## was used. This gives the usual Klein-Gordon field equations ##\partial_{a}\partial^{a}\varphi - m^{2}\varphi = 0 ##. The calculation for the electromagnetic field is extremely similar; you just have a 4-potential ##A_{b}## instead of a scalar potential ##\varphi## so that for example ##\frac{\partial (\partial_{a}A_{b})}{\partial (\partial_{c}A_{d})} = \delta^{c}_{a}\delta^{d}_{b}##.
 
  • #3
You're making things more complicated than they are. And you should use indices consistently:

[tex] L= -\frac{1}{16\pi} F_{\mu\nu}F^{\mu\nu}= -\frac{1}{16\pi}\left(\partial_{\mu}A_{\nu}-\partial_{\nu}A_{\mu}\right)(...) [/tex]

So

[tex]\frac{\partial L}{\partial A_{\alpha}} = 0 [/tex]

and

[tex]\frac{\partial L}{\partial \left(\partial_{\beta}A_{\alpha}\right)} = \left[\frac{1}{8\pi}\frac{\partial}{\partial \left(\partial_{\beta}A_{\alpha}\right)} \left(\partial_{\mu}A_{\nu}-\partial_{\nu}A_{\mu}\right)\right] (...) [/tex]

Now use

[tex] \frac{\partial\left(\partial_{\mu}A_{\nu}\right)}{\partial \left( \partial_{\beta}A_{\alpha}\right)} = \delta^{\beta}_{\mu}\delta^{\alpha}_{\nu} [/tex]
 
  • #4
WannabeNewton said:
Perhaps a simpler example will get you up to speed. Consider the Klein-Gordon Lagrangian density in flat space-time ##\mathcal{L} = -\frac{1}{2}(\eta^{ab}\partial_{a}\varphi \partial_{b}\varphi + m^2 \varphi^2)##.

We have ##\frac{\partial
\mathcal{L}}{\partial \varphi} = -m^{2}\varphi## and ##\frac{\partial\mathcal{L}}{\partial(\partial_{a} \varphi)} = -\frac{1}{2}\{\eta^{bc}\partial_{b}\varphi \frac{\partial (\partial_{c}\varphi)}{\partial (\partial_{a}\varphi)} + \eta^{bc}\partial_{c}\varphi \frac{\partial (\partial_{b}\varphi)}{\partial (\partial_{a}\varphi)}\}\\ = -\frac{1}{2}\{\eta^{bc}\partial_{b}\varphi\delta^{a}_{c} + \eta^{bc}\partial_{c}\varphi \delta^{a}_{b}\}\\ = -\partial^{a}\varphi##
where ##\frac{\partial (\partial_{a}\varphi)}{\partial (\partial_{b}\varphi)} = \delta^{b}_{a}## was used. This gives the usual Klein-Gordon field equations ##\partial_{a}\partial^{a}\varphi - m^{2}\varphi = 0 ##. The calculation for the electromagnetic field is extremely similar; you just have a 4-potential ##A_{b}## instead of a scalar potential ##\varphi## so that for example ##\frac{\partial (\partial_{a}A_{b})}{\partial (\partial_{c}A_{d})} = \delta^{c}_{a}\delta^{d}_{b}##.

aah, thanks.

I was wondering if I get 2 ##\delta##'s.

What should the derivative in the Lagrangian look like (to be compatible with the ##\mathcal{L}## terms)?

It shouldn't contain any indices, which are used inside the ##\mathcal{L}##, right?

In my example ##\frac{\partial A^{\alpha}_{\ ,\nu}}{\partial A^{\nu}_{\ ,\mu}}##

I should get ##\delta^{\alpha}_{\nu} \delta^{\mu}_{\nu}##, right?

This does not look right, so I think I have to use something like ##\frac{\partial \mathcal{L}}{\partial A^{\sigma}_{\ ,\tau}}## ?

dextercioby said:
You're making things more complicated than they are. And you should use indices consistently:

[tex] L= -\frac{1}{16\pi} F_{\mu\nu}F^{\mu\nu}= -\frac{1}{16\pi}\left(\partial_{\mu}A_{\nu}-\partial_{\nu}A_{\mu}\right)(...) [/tex]

So

[tex]\frac{\partial L}{\partial A_{\alpha}} = 0 [/tex]

and

[tex]\frac{\partial L}{\partial \left(\partial_{\beta}A_{\alpha}\right)} = \left[\frac{1}{8\pi}\frac{\partial}{\partial \left(\partial_{\beta}A_{\alpha}\right)} \left(\partial_{\mu}A_{\nu}-\partial_{\nu}A_{\mu}\right)\right] (...) [/tex]

Now use

[tex] \frac{\partial\left(\partial_{\mu}A_{\nu}\right)}{\partial\left(\partial_{\beta}A_{\alpha}\right)} = \delta^{\beta}_{\mu}\delta^{\alpha}_{\nu} [/tex]

And in the 2nd part just the opposite?

[tex] \frac{\partial\left(\partial^{\mu}A^{\nu}\right)}{\partial \left(\partial^{\beta} A^{\alpha}\right)} = \delta^{\mu}_{\beta} \delta^{\nu}_{\alpha} [/tex]
 
  • #5
The nu's must be on diagonal, if summed over.
 
  • #6
Using a slightly different Euler-Lagrange Equation:

##\frac{\partial}{\partial x^\tau} \frac{\partial \mathcal{L}}{\partial A^{\sigma}_{\ ,\tau}} - \frac{\partial \mathcal{L}}{\partial A^\sigma} = 0##

Using this Lagrangian:

##\mathcal{L} = -\frac{1}{16\pi} (\eta_{\mu\alpha} A^{\alpha}_{\ ,\nu} - \eta_{\alpha\nu} A^{\alpha}_{\ ,\mu}) (\eta^{\beta\nu} A^{\mu}_{\ ,\beta} - \eta^{\beta\mu} A^{\nu}_{\ ,\beta})##

Then ##\frac{\partial \mathcal{L}}{\partial A^\sigma} = 0##

Now for example:

##-\frac{1}{16\pi} \eta_{\mu\alpha} \eta^{\beta\nu} \frac{\partial}{\partial A^{\sigma}_{\ ,\tau}} (A^{\alpha}_{\ ,\nu} A^{\mu}_{\ ,\beta})##

which leads to

##-\frac{1}{16\pi} \eta_{\mu\alpha} \eta^{\beta\nu} A^{\alpha}_{\ ,\nu} (\frac{\partial}{\partial A^{\sigma}_{\ ,\tau}} A^{\mu}_{\ ,\beta})##

and

##-\frac{1}{16\pi} \eta_{\mu\alpha} \eta^{\beta\nu} (\frac{\partial}{\partial A^{\sigma}_{\ ,\tau}} A^{\alpha}_{\ ,\nu}) A^{\mu}_{\ ,\beta}##

where

##(\frac{\partial}{\partial A^{\sigma}_{\ ,\tau}} A^{\mu}_{\ ,\beta}) = \delta^{\mu}_{\sigma} \delta^{\tau}_{\beta}##

and

##(\frac{\partial}{\partial A^{\sigma}_{\ ,\tau}} A^{\alpha}_{\ ,\nu}) = \delta^{\alpha}_{\sigma} \delta^{\tau}_{\nu}##
 
  • #7
ProfDawgstein said:
[...]
And in the 2nd part just the opposite?

[tex] \frac{\partial\left(\partial^{\mu}A^{\nu}\right)}{\partial \left(\partial^{\beta} A^{\alpha}\right)} = \delta^{\mu}_{\beta} \delta^{\nu}_{\alpha} [/tex]

Yes, using the partial notation ∂ for derivatives is better, it has more visibility.
 
  • #8
dextercioby said:
Yes, using the partial notation ∂ for derivatives is better, it has more visibility.

Now I ended up with

##-4 k A_{\sigma}^{\ ,\tau} + 3 k A^{\tau}_{\ ,\sigma} + k A^{\mu}_{\ ,\mu}##

##k = \frac{1}{16\pi}##

where the last term looks like a divergence.

I wonder if this is the correct path...
 
  • #9
I must echo dexter here and say that you are making this much more complicated than it needs to be. I'm going to drop the ##4\pi## in the denominator and just write ##\mathcal{L} = -\frac{1}{4}F^{\mu\nu}F_{\mu\nu} + A_{\mu}j^{\mu}## (your exercise does it for the case of vanishing 4-current i.e. ##j^{\mu} = 0## but there's no real difference in the computation regardless). Hence ##\frac{\partial \mathcal{L}}{\partial A_{\nu}} = j^{\nu}##. Now ##\frac{\partial \mathcal{L}}{\partial (\partial_{\mu}A_{\nu})} = -\frac{1}{4}\eta^{\alpha \rho}\eta^{\beta \sigma}\{\frac{\partial F_{\alpha \beta}}{\partial (\partial_{\mu}A_{\nu})}F_{\rho\sigma} +\frac{\partial F_{\rho\sigma}}{\partial (\partial_{\mu}A_{\nu})}F_{\alpha\beta} \}## and ##F_{\alpha\beta} = 2\partial_{[\alpha}A_{\beta]}## so just evaluate.
 
Last edited:
  • #10
WannabeNewton said:
I must echo dexter here and say that you are making this much more complicated than it needs to be. I'm going to drop the ##4\pi## in the denominator and just write ##\mathcal{L} = -\frac{1}{4}F^{\mu\nu}F_{\mu\nu} + A_{\mu}j^{\mu}## (your exercise does it for the case of vanishing 4-current i.e. ##j^{\mu} = 0## but there's no real difference in the computation regardless). Hence ##\frac{\partial \mathcal{L}}{\partial A_{\nu}} = j^{\nu}##. Now ##\frac{\partial \mathcal{L}}{\partial (\partial_{\mu}A_{\nu})} = \eta^{\alpha \rho}\eta^{\beta \sigma}\{\frac{\partial F_{\alpha \beta}}{\partial (\partial_{\mu}A_{\nu})}F_{\rho\sigma} +\frac{\partial F_{\rho\sigma}}{\partial (\partial_{\mu}A_{\nu})}F_{\alpha\beta} \}## and ##F_{\alpha\beta} = 2\partial_{[\alpha}A_{\beta]}## so just evaluate.

I failed on one index...

Using this Euler-Lagrange:

##\frac{\partial}{\partial x^\tau} \frac{\partial \mathcal{L}}{\partial A^{\sigma}_{\ ,\tau}} - \frac{\partial \mathcal{L}}{\partial A^\sigma} = 0##

Now I have

##4 k A^{\tau}_{\ ,\sigma} - 4 k A_{\sigma}^{\ ,\tau}##

which looks a lot better.

Now I just raise the ##\sigma## index with ##\eta^{\sigma\alpha}##

then I get ##4 k ( A^{\tau ,\alpha} - A^{\alpha ,\tau} )##

which is ##4 k F^{\alpha\tau}##

doing ##\partial_\tau## from Euler-Lagrange

leads to ##4 k \partial_\tau F^{\alpha\tau} = 0##.

-----------------------------------------

I just wanted to try my way :)

ProfDawgstein said:
##[\partial_\nu] j^\nu = 0##

I guess he means charge conservation, but wrote ##j^\nu=0##.

Correction: He meant "current-free"...
 
Last edited:
  • #11
Well as long as it worked out :)
 
  • #12
WannabeNewton said:
Well as long as it worked out :)

Two pages and 400 indices later, yes.
It was a total mess, especially that little index mistake :D

NOW I will try your way.

WannabeNewton said:
I must echo dexter here and say that you are making this much more complicated than it needs to be. I'm going to drop the ##4\pi## in the denominator and just write ##\mathcal{L} = -\frac{1}{4}F^{\mu\nu}F_{\mu\nu} + A_{\mu}j^{\mu}## (your exercise does it for the case of vanishing 4-current i.e. ##j^{\mu} = 0## but there's no real difference in the computation regardless). Hence ##\frac{\partial \mathcal{L}}{\partial A_{\nu}} = j^{\nu}##. Now ##\frac{\partial \mathcal{L}}{\partial (\partial_{\mu}A_{\nu})} = \eta^{\alpha \rho}\eta^{\beta \sigma}\{\frac{\partial F_{\alpha \beta}}{\partial (\partial_{\mu}A_{\nu})}F_{\rho\sigma} +\frac{\partial F_{\rho\sigma}}{\partial (\partial_{\mu}A_{\nu})}F_{\alpha\beta} \}## and ##F_{\alpha\beta} = 2\partial_{[\alpha}A_{\beta]}## so just evaluate.

Where

##\frac{\partial F_{\rho\sigma}}{\partial (\partial_{\mu}A_{\nu})} = \frac{\partial (\partial_\rho A_\sigma - \partial_\sigma A_\rho )}{\partial (\partial_{\mu}A_{\nu})} = \delta^{\mu}_{\rho} \delta^{\nu}_{\sigma} - \delta^{\mu}_{\sigma} \delta^{\nu}_{\rho}##

and

##\frac{\partial F_{\alpha\beta}}{\partial (\partial_{\mu}A_{\nu})} = \frac{\partial (\partial_\alpha A_\beta - \partial_\beta A_\alpha )}{\partial (\partial_{\mu}A_{\nu})} = \delta^{\mu}_{\alpha} \delta^{\nu}_{\beta} - \delta^{\mu}_{\beta} \delta^{\nu}_{\alpha}##

Somehow my lazyness prevents me from finishing this :D

This is indeed much nicer, thanks!

Thanks to you both!

[Another apple of knowledge thrown at me by Newton...]
 
Last edited:
  • #13
Don't worry about doing it another way; if you got it once then it's fine. If you're interested: ##\eta^{\alpha \rho}\eta^{\beta \sigma}\{\frac{\partial F_{\alpha \beta}}{\partial (\partial_{\mu}A_{\nu})}F_{\rho\sigma} +\frac{\partial F_{\rho\sigma}}{\partial (\partial_{\mu}A_{\nu})}F_{\alpha\beta} \}\\ = 2\eta^{\rho [\mu}\eta^{\nu]\sigma}F_{\rho\sigma} + 2\eta^{\alpha [\mu}\eta^{\nu]\beta}F_{\alpha\beta} = 4F^{\mu\nu}##
so ##\frac{\partial \mathcal{L}}{\partial (\partial_{\mu}A_{\nu})} = -F^{\mu\nu}## hence ##\partial_{\mu}F^{\nu\mu} = j^{\nu}##.
 
  • #14
WannabeNewton said:
Don't worry about doing it another way; if you got it once then it's fine. If you're interested: ##\eta^{\alpha \rho}\eta^{\beta \sigma}\{\frac{\partial F_{\alpha \beta}}{\partial (\partial_{\mu}A_{\nu})}F_{\rho\sigma} +\frac{\partial F_{\rho\sigma}}{\partial (\partial_{\mu}A_{\nu})}F_{\alpha\beta} \}\\ = 2\eta^{\rho [\mu}\eta^{\nu]\sigma}F_{\rho\sigma} + 2\eta^{\alpha [\mu}\eta^{\nu]\beta}F_{\alpha\beta} = 4F^{\mu\nu}##
so ##\frac{\partial \mathcal{L}}{\partial (\partial_{\mu}A_{\nu})} = -F^{\mu\nu}## hence ##\partial_{\mu}F^{\nu\mu} = j^{\nu}##.

Actually I am working on it right now [hahah], but thanks :)

EDIT:

done

##\partial_\mu F^{\mu\nu} = 0##

looks good
 
  • #15
I was wondering about the whole thing using ##\delta## notation...

Using the Lagrangian

##\mathcal{L} = -\frac{1}{4}F^{\mu\nu}F_{\mu\nu}##

where

##\delta F_{\mu\nu} = \delta (\partial_\mu A_\nu - \partial_\nu A_\mu) = \delta (\partial_\mu A_\nu) - \delta (\partial_\nu A_\mu) = \partial_\mu (\delta A_\nu) - \partial_\nu (\delta A_\mu)##

Now the action

##I = \int \mathcal{L} \ d^4x##

The variation of the action

##\delta I = \int \delta (-\frac{1}{4}F^{\mu\nu}F_{\mu\nu}) d^4x = 0##

where ##F_{\mu\nu}## goes to zero at the surface of the integration volume. [?]

Using the 'product rule'

##\delta I = \int -[(\frac{1}{4} (\delta F^{\mu\nu}) F_{\mu\nu}) + (\frac{1}{4} (\delta F_{\mu\nu}) F^{\mu\nu})] d^4x = 0##

where

##\delta F^{\mu\nu} = \delta (\partial^\mu A^\nu - \partial^\nu A^\mu) = \delta (\partial^\mu A^\nu) - \delta (\partial^\nu A^\mu) = \partial^\mu (\delta A^\nu) - \partial^\nu (\delta A^\mu)##

now plug in, do some multiplication and then partial integration, that's it?

The remaining term should be the field equation then?
 
  • #16
See section E.1 of Wald.
 

1. What is the Electromagnetic Field Lagrangian?

The Electromagnetic Field Lagrangian is a mathematical function used in classical field theory to describe the dynamics of electromagnetic fields. It is based on the principle of least action, which states that the path a system takes between two points is the one that minimizes the action, or the integral of the Lagrangian over time. In simple terms, it is a way to describe the behavior of electromagnetic fields using a single equation.

2. What are the field equations for the Electromagnetic Field Lagrangian?

The field equations for the Electromagnetic Field Lagrangian are known as Maxwell's equations. They consist of four equations, two of which describe the behavior of electric fields and the other two describe the behavior of magnetic fields. These equations are fundamental to understanding the behavior of electromagnetic fields and are used in various fields including optics, electronics, and electromagnetism.

3. How are the field equations derived from the Electromagnetic Field Lagrangian?

The field equations are derived by applying the principle of least action to the Lagrangian. This involves using calculus of variations to find the path that minimizes the action, which results in the four Maxwell's equations. The derivation is complex and involves advanced mathematical techniques, but the end result is a set of equations that accurately describe the behavior of electromagnetic fields.

4. What is the significance of the Electromagnetic Field Lagrangian in physics?

The Electromagnetic Field Lagrangian is significant because it provides a unified framework for understanding and predicting the behavior of electromagnetic fields. It allows scientists to make precise calculations and predictions about the behavior of electromagnetic waves, and it has been used to develop advanced technologies such as radio, television, and telecommunications. It is also an important concept in the development of quantum field theory.

5. Are there any limitations to the Electromagnetic Field Lagrangian?

Like any mathematical model, the Electromagnetic Field Lagrangian has its limitations. It is based on classical field theory and does not take into account quantum effects, so it cannot fully describe the behavior of electromagnetic fields at the subatomic level. Additionally, it assumes that the fields are continuous and smooth, which may not always be the case in certain situations such as near point charges. However, it remains a powerful tool for understanding and predicting the behavior of electromagnetic fields in many practical applications.

Similar threads

  • Special and General Relativity
Replies
9
Views
540
  • Special and General Relativity
Replies
1
Views
766
  • Special and General Relativity
Replies
2
Views
573
  • Special and General Relativity
Replies
7
Views
1K
  • Special and General Relativity
Replies
10
Views
998
  • Special and General Relativity
Replies
2
Views
841
  • Special and General Relativity
Replies
17
Views
1K
  • Special and General Relativity
Replies
4
Views
1K
  • Special and General Relativity
Replies
3
Views
2K
  • Special and General Relativity
Replies
1
Views
1K
Back
Top